Why is D not incorrect?
My thought process concerning this answer was that, although a hefty handling fee could generate ...
treyyb12 on June 4 at 08:41PM
  • December 2014 LSAT
  • SEC4
  • Q24
1
Reply
But what if the price is reduced but it's still...
Wouldn't it be wrong on the basis that we don't know whether or not the price is the highest on t...
NAM1921 on April 17, 2023
  • December 2014 LSAT
  • SEC4
  • Q23
2
Replies
Control group vs unrepresentative sample
This AC made me nervous because the problem with the conclusion is that it needs a control group ...
AndrewArabie on April 16, 2023
  • December 2014 LSAT
  • SEC4
  • Q18
2
Replies
is the stimulus not intentionally misleading?
If a product is free then why is it charging a handling fee? I ruled out D because I assumed it w...
AndrewArabie on April 16, 2023
  • December 2014 LSAT
  • SEC4
  • Q24
3
Replies
Please Explain
Why is the answer A and not D?
rmkrutz@crimson.ua.edu on April 12, 2021
  • December 2014 LSAT
  • SEC4
  • Q21
7
Replies
Why is E correct
Could anybody please explain why E is correct? I don't understand how it guarantees the conclusio...
filozinni on October 21, 2020
  • December 2014 LSAT
  • SEC4
  • Q16
1
Reply
Why is B incorrect?
I was stuck between B & E
aseikhon11 on September 23, 2020
  • December 2014 LSAT
  • SEC4
  • Q17
1
Reply
December 2014 LSAT lr 12
Can someone explain this question in detail? Thanks!
kens on June 4, 2020
  • December 2014 LSAT
  • SEC4
  • Q12
1
Reply
December 2014 SEC 4 Q18
Just out of curiosity, in what sort of situation would answer A be correct? Thanks in advance!
kens on June 4, 2020
  • December 2014 LSAT
  • SEC4
  • Q18
1
Reply
December 2014 SEC 4 Q24
Can someone please explain why answer B is incorrect? Thanks!
kens on June 4, 2020
  • December 2014 LSAT
  • SEC4
  • Q24
1
Reply
explanation please
for the question step and also the answer choices thank you.
Masada on May 14, 2020
  • December 2014 LSAT
  • SEC4
  • Q20
2
Replies
Explanation please
hello can I get a break down of the question and also the answer choices. ( B and C )
Masada on May 14, 2020
  • December 2014 LSAT
  • SEC4
  • Q22
2
Replies
Why is B wrong?
Confused. I just need explaining.
isorom19 on May 13, 2020
  • December 2014 LSAT
  • SEC4
  • Q22
1
Reply
confused
Confused as to why B doesnt work. Thanks!
Ceci on February 14, 2020
  • December 2014 LSAT
  • SEC4
  • Q11
2
Replies
test
test
Patrick-Polsin on February 4, 2020
  • December 2014 LSAT
  • SEC4
  • Q19
2
Replies
C
Why would answering this question require us to infer that there is indeed another reason to lowe...
tomgbean on January 15, 2020
  • December 2014 LSAT
  • SEC4
  • Q15
2
Replies
A and C
Why doesn't A and C work for this passage?
tomgbean on January 15, 2020
  • December 2014 LSAT
  • SEC4
  • Q4
1
Reply
Why is C wrong?
I thought one of the premises was that objectivity is necessary. So if jurors are unable to make ...
manvir on January 14, 2020
  • December 2014 LSAT
  • SEC4
  • Q17
2
Replies
E and Sample size
Why is the sample size unrepresentative?
tomgbean on January 14, 2020
  • December 2014 LSAT
  • SEC4
  • Q18
1
Reply
Why is B correct? Why is C incorrect?
Thanks
Ryan-Mahabir on January 1, 2020
  • December 2014 LSAT
  • SEC4
  • Q26
2
Replies